You are on page 1of 18

“Success consists of going from failure to

failure without loss of enthusiasm."


…Winston Churchill

2
CHAPTER

Linear Time Invariant


(LTI) Systems

Level-1 (C) Non-causal and stable.


(D) Non-causal and unstable.
1. If the lengths of two discrete time sequences
x1 (n) and x2 (n) are 5 and 7 respectively then, 8. If the response of LTI continuous time system
the length of a sequence x1 (n) ∗ x2 (n) is 1 1
for unit step input is ( − e−2t ) , then
(A) 5 (B) 6 2 2
(C) 7 (D) 11 impulse response of the system is
1 1
(A) ( − e−2t) ) (B) e−2t
2. If an LTI system has an impulse response of 2 2
h[n] = αn u[n] then, the condition on α for (C) (1 − e−2t ) (D) Constant
which the system to be BIBO stable is
(A) α > 0 (B) |α| > 1 9. Given that two systems having impulse
(C) |α| < 1 (D) None of these responses h1 [n] and h2 [n] are in cascade. If
h1 [n] is causal, then comment on h[n] where
3. A discrete-time LTI system has an input x[n], h[n] = h1 [n] ∗ h2 [n].
impulse response h[n] and output y[n] (A) h[n] is Causal if h2 [n] is Causal
if x[n] = {1, 3, 4} and h[n] = {1, 2} (B) h[n] is Causal for any h2 [n]
↑ ↑ (C) h[n] is Anti-causal if h2 [n] = δ[n − 3]
then the values of y[0] and y[2] are (D) None of these
___________respectively.
(A) 8 and 5 (B) 8 and 8 10. Consider the discrete time signal
(C) 5 and 8 (D) 5 and 5 x(n) = {1, 1, 1, 1, 0.5, 0.5}
y(n) = δ(n − 1) ∗ x(n) is
4. Convolution of e−5t u(t − 5) with δ(t + 7) is (A) 1 (B) δ(n − 1)
(A) e−35 e−5t u(t − 12)
(C) x(n − 1) (D) 5
(B) e−35 e−5t u(t + 2)
(C) e−30 e−5t u(t − 5)
11. The impulse response of a LTI system is given
(D) e−35 e−5t u(t − 2)
as, h[n] = e−n sin(π⁄6n) u[n] then the system
is
5. A continuous time signal is expressed as
(A) Causal and stable
g(t) = 4 sin(πt⁄8) ∗ δ(t − 3),
(B) Causal and unstable
the value of g(−1) is __________ .
(C) Non-Causal and unstable
6. Convolution of x(t + 5) with impulse signal (D) Non-Causal and stable
δ(t − 7) is equal to
(A) x(t − 2) (B) x(t + 12) 12. The result of the convolution
3
(C) x(t − 12) (D) x(t + 2) x(2t) ∗ δ (3t + ) is
2
1
7. The impulse response of a LTI system is given (A) x[6t + 3] (B) x[2t + 3]
as h[n] = δ[n] + u[n + 2] − u[n − 7]. Then the 3
system is 1 1
(C) x(2t + 1) (D) x[3t + 3]
(A) Causal and stable. 3 3
(B) Causal and unstable.

info@thegateacademy.com ©Copyright reserved. Web:www.thegateacademy.com 20


Linear Time Invariant (LTI) Systems

13. A system is defined by its impulse 20. Consider an LTI system containing a passive
response h(n) = 5n u(−n + 5), the system is electrical network. If the input is a sinusoidal
(A) Stable and causal signal, then the steady state output of the
(B) Unstable and non-causal network is
(C) Stable but non-causal (A) Sinusoidal with the same amplitude,
(D) Causal but unstable frequency and phase
(B) Sinusoidal with the same frequency, but
14. A discrete-time signal x[n] is given by possibly different amplitude and phase
x[n] = {3, 4, 2, 1, 5} (C) Non-sinusoidal
↑ (D) Sinusoidal with a different frequency
if signal y(n) = 5δ(n + 2) ∗ x(n), then
the value of y(n) at n = −3 is__________. Level-2

15. A discrete-time LTI system is having impulse 1. The response of a linear time-invariant
response h[n] = {1, −1} and the discrete-time system to a unit step input u(n)
↑ is the unit impulse δ(n). The system response
input of system is x[n] = {1, 2, 3} to a ramp input n u(n) would be
↑ (A) u(n) (B) u(n − 1)
then, the output of the system is ∞
(A) {1, 1, 1, −3} (C) nδ(n) (D) ∑ kδ (n − k)
↑ k=0
(B) {1, 1, 1, −3}
↑ 2. Given that
(C) {3, 5, 3} x1 (t) = ek1t u(t) and x2 (t) = e−k2 t u(t).
↑ Which one of the following gives their
(D) {1, 3, 1, 1} convolution?
↑ [ek1t − e−k2 t ]
(A)
[k1 + k 2 ]
16. Consider a discrete-time LTI system having
[ek1t − e−k2 t ]
input and impulse response are (B)
[k 2 − k1 ]
x[n] = δ[n] + δ[n − 1]
h[n] = δ[n] − δ[n − 1] [ek1t + e−k2 t ]
(C)
The output of the system will be given by [k 2 + k1 ]
(A) δ[n] − δ[n − 2] [ek1t + e−k2 t ]
(B) δ[n] − δ[n − 1] (D)
[k 2 − k1 ]
(C) δ[n − 1] + δ[n − 2]
(D) δ[n] + δ[n − 1] + δ[n − 2]
3. For a LTI system the input is
x(t) = 3δ(t + 2) − 2δ(t) + 5δ(t − 3).
17. The impulse response of a system is
The impulse response h(t) is shown in figure.
h(t) = δ(t − 0.5). If two such systems are
cascaded, the impulse response of the overall h(t)
system will be 1
(A) 0.5 δ(t − 0.25) (B) δ(t − 0.25)
(C) δ(t − 1) (D) 0.5 δ(t − 1) 0 t
The output y(t) at t = 4 is __________.
18. The impulse response of a continuous time
system is given by h(t) = δ(t + 3) − δ(t − 3). 4. A LTI system of impulse response,
The value of step response at t = 0 is __________. h(t) = e−αt u(t); α ≥ 0 is given an input of
x(t) = u(t). Then, the output of the LTI system
19. The value of 2δ(t) ∗ 4δ(t − 4) ∗ 5δ(t − 3) is is
(A) 40δ(t + 7) (B) 40δ(t − 7) 1
(C) 20δ(t − 4) (D) 20δ(t − 5) (A) [1 + e−αt ]u(t)
α
info@thegateacademy.com ©Copyright reserved. Web:www.thegateacademy.com 21
Linear Time Invariant (LTI) Systems
1 (C) r(t − 3) (D) r(t − 8)
(B) [1 − e−αt ]u(t)
α
(C) α[1 − αe−αt ]u(t) 10. x(t) and h(t) of a LTI system are shown in
figure below.
(D) α[1 − e−αt ]u(t)
x(t) h(t)

5. Consider the cascade structure of LTI systems


1
shown in figure. 1 0 1
t t
h2 (t)
−1 −1
x(t) h1 (t) + y(t) The output of the system y(t) is
h3 (t) (A) y(t)

h(t) 1

If h1 (t) = u[t + 3] 0 1
t
h2 (t) = 3δ[t − 2] 2
h3 (t) = 2δ[t + 1]
−1
The value of h(3) is __________.
(B) y(t)
6. For an input x(t) = e u(t), the output of
−2(t−3)
−1 0 t
a system having impulse response
h(t) = e−t u(t) will be
−1
(A) e6 (e−t −e−2t )u(t)
(C) y(t)
(B) e6 (e−2t +e−t )u(t)
(C) e6 (−e−t −e−2t )u(t) 1
6
e 0
(D) (e−t −e−2t )u(t) t
3 −1
(D) None of these
7. The unit sample response of a discrete system
1 1
is 1 0 0 0 … For an input sequence 11. The output of an LTI system for an input of
2 4
1 0 1 0 0 0…, the corresponding output x[n] = {1, 1} is {1, 2, 1}. Then impulse response of
sequence is ↑ ↑
1 1 1 1 the LTI system is
(A) 1 0 0… (A) {1, 1}
2 4 2 4
1 ↑
(B) 1 0 0 0… (B) {1, 1}
4

1 5
(C) 2 0 0… (C) {1, 0, 1}
2 4

1 5 1 1
(D) 1 0 0… (D) {1, 0, 1}
2 4 2 4 ↑
8. A discrete time signal x(n) is {1, 3, 4, 2}
12. The input x(t) and impulse response h(t) of an

LTI system shown in figure are
The value of y(n) = x(2n) ∗ x(n) at n = 3 is
x(t) y(t)
___________. h(t)

9. A LTI system has impulse response of x(t) = δ(t − 1) − 2δ(t + 1)


u(t − 5). If u(t − 3) is given as input to the h(t) = u(t + 1) − u(t − 3)
system, then the output is The waveform which corresponds to output
(A) u(t − 8) (B) r(t + 5) y(t) is

info@thegateacademy.com ©Copyright reserved. Web:www.thegateacademy.com 22


Linear Time Invariant (LTI) Systems
(A) y(t) 15. The graph shown below represents a
−2 0 2 4 waveform obtained by convolving two
t
−1 rectangular waveforms of duration,
y(t)
−2

Amplitude
Figure − I
(B) y(t)

1 t
−2 −4 −2 0 2 4
0
2 t Time
4
−1 (A) Four units each
−2 (B) Four and two units respectively
(C) Six and three units respectively
(C) y(t) (D) Six and two units respectively
3
16. In the LTI system shown below
2
x(t) = u(t − 2)and h(t) = u(t − 4). The plot
1 for output y(t) will be
t h(t) y(t)
−2 0 2 4 x(t)
(D) y(t)
(A) y(t)
3 1
2 t
0 6
(B) y(t)
−2 0 t
2 4 1

13. A LTI system is shown in figure below.


t
x(t) h(t) y(t) 0 6
[Input] [Output] (C) y(t)
Let x(t) = u(t)
h(t) = 3δ(t + 3) − 2δ(t + 1) + 4δ(t − 2) − 5δ(t − 4) 1
The energy of y(t) is ____________ J.
0 t
6 7
14. A discrete time LTI system is shown in figure. (D) y(t)
h(n) y(n)
x(n)
1
The input and output are given by
x[n] = {2, 3 5} and y[n] = {2, −1, −1, −10} t
0 2 3
↑ ↑
respectively. The impulse response h(n) is
(A) {1, 3, 2} 17. The impulse response of a linear time
↑ invariant system is h(n) = {1, 2, 1, −1}. The
(B) {1, −2} response for the input signal x(n) = {1, 2, 3, 1}
↑ is
(C) {1, −2, 1} (A) {1, 8, 4, 8, 3, −1, −2}
↑ (B) {1, 4, 8, 3, 8, −2, −2}
(D) {1, −2} (C) {1, 4, 8, 8, 3, −2, −1}
↑ (D) {1, 8, 3, 8, 8, 4, −1}

info@thegateacademy.com ©Copyright reserved. Web:www.thegateacademy.com 23


Linear Time Invariant (LTI) Systems

18. The impulse response of a LTI system is given 2. A continuous time LTI system is described by
as h[n]. If system is non-causal and stable, t+
T
2
then which of the following could be the LTI 1
y(t) = T{x(t)} = ∫ x(τ) dτ
system? T
T
t−
1 n 2
(A) ( ) u[n] The impulse response is
2
(B) − 2n u[−n − 1] (A) h(t)
(C) 3n u[n] 1/T
(D) None of these
t
−T/2 0 T/2
19. A system with an input x[n] and output y[n] is (B) h(t)
characterized by following equation
2 1/T
1
y[n] = ∑ (3 − k) x[n − k] t
6 −T 0 T
k=0
The impulse response of the system is: (C) h(t)
1 T
(A) h[n] = {1, 2, 3}
6
↑ t
−T/2 0 T/2
1
(B) h[n] = {3, 3, 3} (D) h(t)
6
↑ T
1
(C) h[n] = {1, 1, 1} t
6 −T 0 T

1
(D) h[n] = {3, 2, 1} 3. The impulse response of a LTI system is given
6
↑ as h(t) = u(t) − u(t − T).
If x(t) = h(t) is the input to system, then
+∞
20. For an LTI system input signal x(t) and
∫ y(t) is
impulse response h(t) is shown below. −∞
x(t) h(t) (A) T 2 /2 (B) T 2
(C) T 2 /4 (D) None of these
5
1
4. The input and output sequence of a system are
2
given as
t t
0 4 6 0 x(n) = {3, −5, 2, 4} and

The value of output y(t) at t = 8 sec is _________.
y(n) = {15, −19, 12, 4, 16, 16}
10↑
Level-3 TT
3 ⟹
3
Which of the 10
3
following statements are true
about above system?

3 T3
1. The sequences
(i) System is causal10and stable
x(n) = (a)n u(n) and y(n) = (b)n u(n), where ⟹
(ii) System is non-causal
3 and stable
u(n) is the unit step sequence. The T2
(iii) System has impulse response
convolution of x(n) and y(n) is equal to z(n). 3
h(n) ⇒⟹{5,5 2, 4}
z(n) = x(n) ∗ y(n) T2
↑ 3
If (i) if a = 3 and b = 2 then z(2) = k1 ⟹
(iv) The energy of input
5 signal is
(ii) if a = 2 and b = 2 then z(2) = k 2 .
E ⇒ 54 units
Then the value of k1 + k 2 is ___________.
(A) Only (i), (ii) and (iv) are correct
(B) Only (ii), (iii) and (iv) are correct
(C) Only (i), (ii) and (iii) are correct
(D) All statements are correct

info@thegateacademy.com ©Copyright reserved. Web:www.thegateacademy.com 24


Linear Time Invariant (LTI) Systems

5. A continuous time signal x(t) is shown in (D) None of these


figure.
x(t) 8. The input and impulse of a LTI system shown
6 below are given as
5 x(t) = e−at u(t); h(t) = e−bt u(t)
2 x(t) h(t) y(t)
t The value of output y(t)
−4 −2 0 1 3 5
(i) if a = 2 and b = 5, then y(t) ⇒ y1 (t)
−4 (ii) if a = 5 and b = 5 then y(t) ⇒ y2 (t)
−9 The value of y1 (t) and y2 (t) are respectively
t
__________.
Let y(t) = ∫ x(t) dt. e2t − e−5t
−∞ (A) y1 (t) ⇒ u(t)
∞ 3
−2t
y2 (t) ⇒ t e u(t)
The value of ∫ y(t) dt is ___________.
−∞ e−5t − e−5t
(B) y1 (t) ⇒ u(t)
2
6. Consider two DT systems S1 and S2 described y2 (t) ⇒ t e−5t u(t)
by their impulse responses e−5t − e−3t
S1 : h[n] = e−2|n| (C) y1 (t) ⇒ ( ) u(t)
2
S2 : h[n] = 2n u[n − 1] y2 (t) ⇒ t 2 e−5t u(t)
The following statements are given regarding
e−2t − e−5t
the stability and causality of the systems. (D) y1 (t) ⇒ ( ) u(t)
1. S1 is causal and stable 3
2. S1 is stable but not causal y2 (t) ⇒ t. e−5t u(t)
3. S2 is causal but unstable
4. S2 is causal and stable 9. A continuous time system is shown in figure.
Which of the above statements is/are true? x(t) h(t) y(t)
(A) 1 and 4 (B) 2 and 4
(C) 1 and 3 (D) 2 and 3 x(t)

7. A signal x[n] is passed through a LTI system 4


1 1
shown below. If h1 [n] = { , },
2 2 2
↑ 1
h2 [n] = {1, −1}, h3 [n] = {0, 1, −1} and
↑ ↑ 0 1 2 3 4 t
h4 [n] = δ[n − 1], Find y[n]in terms of x[n]. h(t)
h2 [n]
1
+
x[n] h1 [n] h4 [n] y[n]
+
0 1 t
h3 [n]
The value of y[t + 2] at t = 2.5 is ___________.
1
(A) y[n] = (x[n] − x[n − 1] + x[n − 3]
2 10. Let y(n) = x(n) ∗ h(n),
−x[n − 4])
where x(n) = {2, x, y, 5},
1
(B) y[n] = (x[n − 1] − x[n − 2] + x[n − 3] ↑
2
−x[n − 4]) h(n) = {3, 2, 2}. If y(1) = 7 and y(4) = 12
1 ↑
(C) y[n] = (x[n − 1] + x[n − 2] − x[n − 3] then, the value of y(3) is ___________.
2
−x[n − 4])

info@thegateacademy.com ©Copyright reserved. Web:www.thegateacademy.com 25


Linear Time Invariant (LTI) Systems

11. An inter-connection of LTI systems is shown 15. The signals x(t) and h(t) shown in the figures
in the figure. The impulse responses are x(t)
h1 [n] = 0.5δ[n − 1] + 0.7δ[n] and
1
h2 [n] = δ[n − 1] If the response of overall
system be Aδ[n] + Bδ[n − 1] + Cδ[n − 2],
then A + B + C is___________. −1 0 +1 t
h1 [n]
−1
x[n] + y[n] h(t)

h2 [n] −h1 [n] 1

12. Let z(t) = x1 (t) ∗ x2 (t)


0 1 t
x1 (t) 3
1 If y(t) = f(t) ∗ z(t),
t
dh(t)
where f(t) = ∫ x(τ). dτ and z(t) =
t −∞ dt
1 0 1
− + The energy of signal y(t) is __________.
2 2
x2 (t)
1
16. A continuous time LTI system is shown below
x(t) h(t) y(t)
t
9 0 9 The input signal applied to the system is
− +
2 2 x(t) ⇒ 5 sin 2t + 8 cos 4t + 14 cos[6t + 30°].
The energy of signal z(t) is __________. If the frequency response H(ω) is a
rectangular pulse from ω = −3 to ω = 3 and
13. The input to a matched filter of impulse having amplitude equal to 2 then the power of
response h(t) is given below. output is _________watts.
s(t)
1 17. Consider the cascade structure of LTI systems
shown in figure
2 t
1 3 4 h2 (t)
+ h3 (t) h4 (t) +
x(t) y(t)
⇓ h1 (t) h5 (t)
−1
δ(t)
Slope of h(t) in the interval 3 to 4 and output h(t)
y(t) at t = 1 respectively are If h1 (t) = 4 δ(t + 5)
(A) 0 and 2 (B) 1 and 0.5 h2 (t) = 2 δ(t − 2)
(C) −1 and 0.5 (D) −1 and 0
h3 (t) = 3 u(t − 3)

14. Consider two DT signals given as h4 (t) = 2 δ(t + 1)


x[n] = (0.8)n u[n] h5 (t) = u(t)
and h[n] = (0.4)n u[n] Then the value of y(t) at t = 4.5 is _________.
If y[n] = x[n] ∗ h[n],
then what will be the value of sample y[1]? 18. The impulse response of a continuous time
system is
h(t) = r(t) − r(t − 2) + 2u(t − 3)
−6u(t − 4) + 2r(t − 4) − 2r(t − 5)
{where r(t) is Ramp Signal and u(t) is step
signal}

info@thegateacademy.com ©Copyright reserved. Web:www.thegateacademy.com 26


Linear Time Invariant (LTI) Systems

If input x(t) = u(t) − u(t − 4), then output


y(t) at t = 2 is __________.

19. A continuous time signal x(t) shown in below


x(t)
8
4
2 3
1

−2
−5 −3 −1 0 1 3 4 6 t

−6
−12
−2
Let h(t) = u(t − 2) and
y(t) = x(t) ∗ h(t).
If Area of Signal y(t) is A unit and energy of
signal y(t) is E units then the values of A & E
are respectively.
(A) A = 103 unit & E = 1001 units
(B) A = 93 unit & E = 1098 units
(C) A = 103 unit & E = 1101 units
(D) A = 106 unit & E = 1104 units

20. Consider the following figure


h1 (t)
+
x(t) ∑ h3 (t) y(t)
Input + Output
h2 (t)

h(t)
If h1 (t) = 2δ(t + 3),
h2 (t) = 3δ(t − 1) + δ(t + 2),
h3 (t) = 4u(t − 3).
The value of h(t) at t = 5 and the average
power of output of y(t) if input x(t) = δ(t) are
respectively
(A) 24 and P = 284 units
(B) 24 and P = 288 units
(C) 12 and P = 280 units
(D) 12 and P = 24 units

info@thegateacademy.com ©Copyright reserved. Web:www.thegateacademy.com 27


Linear Time Invariant (LTI) Systems

6. [Ans. A]
Answer keys and Solutions We know that
x(t − t1 ) ∗ δ(t − t 2 ) = x(t − t 2 − t1 )
Level-1 x(t + 5) ∗ δ(t − 7) = x(t − 7 + 5)
= x(t − 2)
1. [Ans. D]
Length of x1 (n) ∗ x2 (n) is 7. [Ans. C]
L = L1 + L2 − 1 h[n] ≠ 0 for n < 0 ⇒ Non­causal
Where L1 is length of x1 (n) [for n = −2, −1]
and L2 is length of x2 (n)
∑ |h[n]| is finite ⇒ Stable
L1 = 5, L2 = 7
n
L =5+7−1
L = 11 8. [Ans. B]
Impulse response
2. [Ans. C] d
As the system is stable = [step response]
dt
+∞
d 1 1
∑ |h[n]| < ∞ = ( − e−2t )
dt 2 2
n=−∞
+∞ ∞
= e−2t
1
∴ ∑ | (α)n u(n)| = ∑|(α)n | =
(1 − |α|) 9. [Ans. A]
n=−∞ n=0
If |α| < 1; then the system will be BIBO stable. If h2 [n] is Causal then only h[n] is Causal.

3. [Ans. C] 10. [Ans. C]


x(n) = {1, 3, 4}, h(n) = {1, 2} y(n) = x(n) ∗ δ(n − 1)
↑ ↑ ⇒ x(n) ∗ δ(n − no ) = x(n − no )
By applying add and shift method y(n) = x(n − 1)

1 2 11. [Ans. A]
We know that
→1 1 2
+ ∑|h[n]| is finite ⇒ Stable
3 3 6
+ n
4 4 8 h[n] = 0 for n < 0 ⇒ Causal

y(n) = {1, 5, 10, 8}


12. [Ans. C]
↑ 3
y(0) = 5 and y(2) = 8 x(2t) ∗ δ [3t + ]
2
1
4. [Ans. B] = x(2t) ∗ δ [3 (t + )]
2
e−5t u(t − 5) ∗ δ(t + 7) 1 1
= x(2t) ∗ δ [t + ]
= e−5(t+7) u(t + 7 − 5) 3 2
= e−35 e−5t u(t + 2) 1 1
= x [2 (t + )]
3 2
1
5. [Ans. *]Range: −𝟒 𝐭𝐨 − 𝟒 = x[2t + 1]
πt 3
g(t) = 4 sin ( ) ∗ δ(t − 3)
8
π(t − 3) 13. [Ans. C]
= 4 sin ( ) h(n) = 5n u(−n + 5)
8
π(−1 − 3) ∑|h[n]| is finite ⇒ Stable
Now, g(−1) = 4 sin ( ) n
8
−π h[n] ≠ 0 for n < 0 ⇒ Non­Causal
= 4 sin ( ) = −4
2

info@thegateacademy.com ©Copyright reserved. Web:www.thegateacademy.com 28


Linear Time Invariant (LTI) Systems

u(−n + 5) 18. [Ans. *]Range 0.99 to 1.01


h(t) = δ(t + 3) − δ(t − 3)
1 t

⋯⋯⋯⋯ s(t) = ∫ h(t) dt ⇒ u(t + 3) − u(t − 3)


0
−1 0 1 2 3 4 5 n
h(n) ⟶ Stable + Non causal s(t)
1
14. [Ans. *]Range: 14.9 to 15.1
Given, y(n) = 5δ(n + 2) ∗ x(n)
= 5x(n + 2) −3 0 t
+3
Given, x(n) = {3, 4, 2, 1, 5} s(0) = 1

then, x(n + 2) = {3 4 2 1 5} 19. [Ans. B]
↑ We know that
Now, y(n) = 5x(n + 2) ⇒ {15, 20, 10, 5, 25} x(t) ∗ δ(t − t o ) = x(t − t o )
↑ By applying, 2δ(t) ∗ 4δ(t − 4) ∗ 5δ(t − 3)

So, y(n) at n = −3 is 15 or y(−3) = 15
8δ(t − 4) ∗ 5δ(t − 3)
= 40δ(t − 3 − 4)
15. [Ans. A]
= 40δ(t − 7)
h[n] = {1, −1} and x[n] = {1, 2, 3}
↑ ↑
20. [Ans. B]

1 2 3 If the input applied to an LTI system is a
1 2 3 sinusoidal signal then the output of the system
1
+ + is also sinusoidal with different amplitude and
→ −1 −1 −2 −3 different frequency.

y[n] = {1, 1, 1, −3} x(t) LTI y(t)



Here x(t) = sin ωt
y(t) = |h(t)|x(t ± θ) = A sin(ωt ± θ)
16. [Ans. A]
x[n] = δ[n] + δ[n − 1]
x[n] = {1,1}
Level-2

h[n] = δ[n] − δ[n − 1] 1. [Ans. B]
h[n] = {1, −1} and u(n) System δ(n)
↑ From the above input and output relation
↓ would be
1 1
y(n) = x(n) − x(n − 1) ⋯ (1)
→1 1 1 If input x(n) = n u(n)
−1 −1 −1 y(n) = n u(n) − (n − 1) u(n − 1)
= n u(n) − n u(n − 1) + u(n − 1)
y[n] = {1, 0, −1} = n {u(n) − u(n − 1)} + u(n − 1)
↑ = n δ(n) + u(n − 1) = u(n − 1)
y[n] = δ[n] − δ[n − 2]
2. [Ans. A]
17. [Ans. C] Convolution of x1 (t)and x2 (t) is given by

From properties
δ(t − t1 ) ∗ δ(t − t 2 ) = δ(t − t 2 − t1 ) y(t) = ∫ x1 (τ) x2 (t − τ) dτ
By applying −∞

δ(t − 0.5) ∗ δ(t − 0.5) = δ(t − 1)

info@thegateacademy.com ©Copyright reserved. Web:www.thegateacademy.com 29


Linear Time Invariant (LTI) Systems

h(t)
= ∫ ek1τ u(τ)e−k2(t−τ) u(t − τ)dτ
5
−∞
Since 2
u(τ) u(t − τ) = 1,
t
for τ > 0 and t − τ > 0 or τ < t −4 −1 3
t
h(3) = 5
So, y(t) = ∫ ek1τ e−k2 t ek2τ dτ
0
6. [Ans. A]
t
Let the output is y(t)
= e−k2 t ∫ e(k1+k2 )τ dτ y(t) = x(t) ∗ h(t)
0
= e−2(t−3) u(t) ∗ e−t u(t)
e(k1+k2)t − 1 = e6 [e−2t u(t) ∗ e−t u(t)]
= e−k2 t [ ]
(k1 + k 2 ) We know that
ek1t − e−k2 t e λ1 t − e λ2 t
= eλ1t u(t) ∗ eλ2t u(t) = u(t), λ1 ≠ λ2
(k1 + k 2 ) λ1 − λ2
Here, λ1 = −2, λ2 = −1
3. [Ans. *]Range: 5.99 to 6.01 e−2t − e−t
So, y(t) = e6 [ ] u(t)
x(t) = 3δ(t + 2) − 2δ(t) + 5δ(t − 3) −2 − (−1)
= e6 [e−t − e−2t ] u(t)
⇒ Output, y(t) = x(t) ∗ h(t)
= h(t) ∗ [3δ(t + 2) − 2δ(t) + 5δ(t − 3)] 7. [Ans. D]
⇒ y(t) = [3h(t + 2) − 2h(t) + 5h(t − 3)] y(n) = x(n) ∗ h(n)
By add and shift method
h(t) = u(t)
h(n)
y(t) ⇒ 3u(t + 2) − 2u(t) + 5u(t − 3) 1 1/2
x(n) 1/4 0 0 0
y(t)
1 1 1/2 1/4 0 0 0
6 + + + +
0 0 0 0 0 0 0
3
+ + +
1 1 1 1/2 1/4 0 0 0
−2 0 3 + +
0 0 0 0 0 0 0
y(4) = 6 +
0 0 0 0 0 0 0
4. [Ans. B] 0 0 0 0 0 0 0
+∞

y(t) = x(t) ∗ h(t) = ∫ x(τ) h(t − τ) dτ Output sequence is


−∞ 1 1 1 1
+∞ y(n) = { 1 (1 + ) 0 0⋯
2 4 2 4
= ∫ u(τ)e−α(t−τ) u(t − τ) dτ
1 5 1 1
−∞ ={1 0 0⋯
t 2 4 2 4
1
= ∫ e−α(t−τ) dτ = (1 − e−αt ) u(t)
α 8. [Ans. *]Range: 17.99 to 18.01
0
x(n) = {1, 3, 4, 2}
5. [Ans. *]Range: 4.99 to 5.01 ↑
Given, h1 (t) = u(t + 3), h2 (t) x(2n) = {1, 4}
h2 (t) = 3δ(t − 2), h3 (t) = 2δ(t + 1) ↑
h(t) = h1 (t) ∗ [h2 (t) + h3 (t)] By add and shift method
h(t) = u(t + 3) ∗ [3δ(t − 2) + 2δ(t + 1)] 1 3 2 4
= 3u(t + 3) ∗ δ(t − 2) + 2u(t + 3) ∗ δ(t + 1) 1 1 3 2 4
h(t) = 3u(t + 1) + 2u(t + 4) + + +
4 4 12 16 8

info@thegateacademy.com ©Copyright reserved. Web:www.thegateacademy.com 30


Linear Time Invariant (LTI) Systems

⇒ {1, 7, 16, 18, 8} h(n) = {1, 1}


↑ ↑
y(3) = 18
12. [Ans. B]
9. [Ans. D] Given that
In LTI system Impulse response,
From property of convolution h(t) = u(t + 1) − u(t − 3)
x(t − α) ∗ h(t − β) = y(t − α − β) Input, x(t) = δ(t − 1) − 2δ(t + 1)
(α and β are constants) Output, y(t) = x(t) ∗ h(t)
u(t) ∗ u(t) = r(t) = [δ(t − 1) − 2δ(t + 1)] ∗ h(t)
Hence, u(t − 3) ∗ u(t − 5) = δ(t − 1) ∗ h(t) − 2δ(t + 1) ∗ h(t)
= r(t − 3 − 5) = r(t − 8) = h(t − 1) − 2h(t + 1)
h(t)
10. [Ans. A]
1
y(t) = x(t) ∗ h(t)
⇒ x(t) = 1δ(t) − 1δ(t − 1) [from figure]
⇒ y(t) = h(t) ∗ x(t) t
−1 0 3
= h(t) ∗ [δ(t) − δ(t − 1)]
h(t − 1)
= [h(t) ∗ δ(t) − h(t) ∗ δ(t − 1)]
⇒ y(t) = h(t) − h(t − 1)
1
h(t)

t
0 1 0 4
t
2h(t + 1)
−1 2
−h(t − 1)
1
⇒ y(t)
1 2 t
1 −2 0
0 t
2 y(t)
0 t
1 2
−1 1
−2 0
2 t
4
11. [Ans. B] −1
In LTI system −2
For x[n] = {1, 1} ⇒ 2 samples (0 ≤ n ≤ 1)

13. [Ans. *]Range: 70 to 72
y[n] = {1, 2, 1} ⇒ 3 smaples (−1 ≤ n ≤ 1)
y(t) = x(t) ∗ h(t)

3δ(t + 3) − 2δ(t + 1)
⇒ h[n] will have 2 samples y(t) = u(t) ∗ [ ]
+4δ(t − 2) − 5δ(t − 4)
(−1 ≤ n ≤ 0)
y(t) = 3u(t + 3) − 2u(t + 1) + 4u(t − 2)
Let h(n) = {a, b}
−5u(t − 4)
1 1
y(t)
a a a 5
+
b b b 3
5
1
y[n] = {a, a + b, b}
↑ −3 −1 0 1 2 4
By comparing with given output {1, 2, 1} E = (3)2 × 2 + (1)2 × 3 + (5)2 × 2
↑ = 9 × 2 + 1 × 3 + 25 × 2
⇒ a = 1 ,b = 1 = 18 + 3 + 50 = 71 J

info@thegateacademy.com ©Copyright reserved. Web:www.thegateacademy.com 31


Linear Time Invariant (LTI) Systems

14. [Ans. B] 17. [Ans. C]


Given LTI system h(n) = [1, 2, 1, −1]
the input is x(n) = [1, 2, 3, 1]
x[n] = {2, 3, 5}; −1 ≤ n ≤ 1 y(n) = x(n) ∗ h(n)
↑ 1 2 3 1
the output is
1 1 2 3 1
y[n] = {2, −1, −1, −10}; −1 ≤ n ≤ 2
+ + +
↑ 2 2 4 6 2
We know limits of output = limits of input + + + +
limits of impulse response 1 1 2 3 1
let limits of h[n]be; a ≤ n ≤ b
+ + +
⇒ a − 1 = −1 and b + 1 = 2 −1 −1 −2 −3 −1
⇒ a = 0 and b = 1
Hence, limits of impulse response are y(n) = [1, 4, 8, 8, 3, −2, −1]
h[n] = {x, y}
↑ 18. [Ans. B]
From options only (B) matching 1 n
( ) u[n] ⇒ Causal and stable
h[n] = {1, −2} 2
↑ −2n u[−n − 1] ⇒ Non­causal and stable
3n u[n] ⇒ Causal and unstable
15. [Ans. D]
(i) When two rectangular waveforms of 19. [Ans. D]
different lengths L1 and L2 are To obtain impulse response take input
convolved, the resultant waveform will x[n] = δ[n]
2
be trapezoidal of length L1 + L2 . 1
h[n] = ∑(3 − k)δ[n − k]
(ii) When two rectangular waveforms of 6
k=0
same length are convolved, the resultant 1
waveform will be triangular of length 2L. = [3δ[n] + 2δ[n − 1] + δ[n − 2]]
6
From figure given in the question, L1 + L2 = 8 1
= {3, 2, 1}
6
16. [Ans. C] ↑
y(t) = x(t) ∗ h(t)

20. [Ans. *]Range: 23.9 to 24.1
= ∫ x(τ) h(t − τ) dτ y(t) = x(t) ∗ h(t)
−∞ h(t) = u(t) → from diagram

y(t) = x(t) ∗ u(t)
= ∫ u(τ − 2) u(t − τ − 4) dτ
dy(t) du(t)
−∞ = x(t) ∗
dt dt
u(τ − 2) = 1, for τ − 2 > 0 or τ > 2 dy(t)
u(t − τ − 4) = 1, for t − τ − 4 > 0, = x(t) ∗ δ(t)
dt
or τ < t − 4 dy(t)
y(t) = x(t)
dt
t
y(t) = ∫ x(t) dt
0
1
x(t)
t
0 6 7
t−4
5

So, y(t) = ∫ (1)(1) dτ 2


2
t
= (t − 4 − 2) u(t − 6) 0 4 6
= (t − 6) u(t − 6)

info@thegateacademy.com ©Copyright reserved. Web:www.thegateacademy.com 32


Linear Time Invariant (LTI) Systems
t
y(t)
y(t) = ∫ x(t) dt
0
T
y(t)
24
0 T 2T t
20
Area under triangle
1
t = 2 × × T × T = T2
0 4 6 2
+∞
y(8) = 24
Then ∫ y(t) is = T 2
−∞
Level-3
4. [Ans. B]
1. [Ans. *]Range: 30.9 to 31.1 Let h[n] = {a, b, c}, then output is
z(n) = (a)n u(n) ∗ (b)n u(n) ↑
(i) If a = 3, b = 2 3 −5 2 4

a 3a −5a 2a 4a
z(n) = ∑ (3)k u(k)(2)n−k u(n − k)
b 3b −5b 2b 4b
k=∞
(3)n+1
− (2)n+1 c 3c −5c 2c 4c
z(n) = ∙ u(n)
3−2
3
(3) − (2) 3 3a = 15 3b − 5a = −19 4c = 16
z(2) ⇒ ∙ u(2) = 27 − 8
1 a=5 b=2 c=4
⇒ k1 = 19 5, 2, 4
h(n) = { }
(ii) If a = 2, b = 2 ↑
z(n) = (2)n (n + 1) ∙ u(n) ⇒ h(n) is finite sequence. So that the system is
z(2) = (2)2 (2 + 1) stable

⇒ k 2 = 12
∵ { ∑ h(n) < ∞}
∴ k1 + k 2 = 31
n=−∞
⇒ h(n) is Non­causal h(n) ≠ 0 n < 0
2. [Ans. A] ⇒ Energy of input signal x(n)
t+T⁄2 ∞
1 2
y(t) = ∫ x(τ) dτ E = ∑ (x(n))
T
t−T⁄2 n=−∞

x(τ) = δ(τ) = (3)2 + (−5)2 + (2)2 + (4)2


y(t) = h(t) = 9 + 25 + 4 + 16
t+T⁄2 = 54
1
h(t) = ∫ δ(τ) dτ
T 5. [Ans. *]Range: 55 to 55
t−T⁄2
1 T T x(t)
h(t) = [u (t + ) − u (t − )] 6
T 2 2
5
2
3. [Ans. B]
h(t) = u(t) − u(t − T) = x(t) t
−4 −2 0 1 3 5
h(t) x(t)
−4
1 1 −9

t t
0 T 0 T

And there convolution y(t) is given as

info@thegateacademy.com ©Copyright reserved. Web:www.thegateacademy.com 33


Linear Time Invariant (LTI) Systems
y(t) e−2t − e−5t
y1 (t) = [ ] u(t)
9 3
7
7 y2 (t) = if a = 5, b = 5
y2 (t) = t e−5t ∙ u(t)
5 +3

5
t 9. [Ans. *]Range: 6.4 to 6.6
−4 −2 1 3
∞ x(t − 1)
∫ y(t) dt = Area of signal y(t) 4
−∞
y(t)Area = (5 × 2) + (7 × 3) + (3 × 2) + (9 × 2)
= 10 + 21 + 6 + 18 = 55 2
1
6. [Ans. D]
0 2 3 4 5 t
System S1 : h[n] = e−2|n|
h[n] ≠ 0, for n < 0 (non causal) y(t)
∞ ∞ 7
−2|n|
∑ |h[n]| = ∑ e 6
n=−∞ n=−∞
−1 ∞
2
= ∑ e2n + ∑ e−2n
n=−∞ n=0
−2 0 2 3 4 5 t
e 1
= −2
+ <∞ y(t) = x(t) ∗ h(t) ; h(t) = u[−t + 1]
1−e 1 − e−2
System is stable and statement 2 is true. dy(t) dh(t)
[ = x(t) ∗ ]
System S2 : h[n] = 2n u[n − 1] dt dt
h[n] = 0, for n < 0 (causal) dy(t)
∞ ∞ = x(t) ∗ δ[−t + 1]
dt
∑ |h[n]| = ∑ |2n u[n − 1]| = [x(t) ∗ δ(t − 1)]
n=−∞

n=−∞ dy(t)
= x[t − 1]
= ∑ 2n = 2 + 4 + 8 … dt
t
n=1 y(t) = ∫ x(t − 1) ∙ dt
This is not absolutely summable, thus the −∞

system is not stable. y[t + 2] at t = 2.5 is (from diagram)


Statement 3 is true. = 6.5

7. [Ans. C] 10. [Ans. *]Range: 18.99 to 19.01


h[n] = h1 [n] ∗ (h2 [n] + h3 [n]) ∗ h4 [n] 2 x y 5
1 1 3 6 3x 3y 15
= { , } ∗ {1, 0, −1} ∗ {0, 1}
2 2 2 4 2x 2y 10
↑ ↑ ↑ 2 4 2x 2y 10
1 1 1 1 1 1 y(n) = {6, 4 + 3x, 4 + 2x + 3y,
= { , } ∗ {0, 1, 0, −1} = {0, , , − , − }
2 2 2 2 2 2 ↑
↑ ↑ ↑ 2x + 2y + 15, 2y + 10, 10}
1 1 1 1 y(1) = 7
y[n] = x[n] ∗ {0, , , − , − }
2 2 2 2 4 + 3x = 7
1
= {x[n − 1] + x[n − 2] − x[n − 3] 3x = 3
2
x=1
− x[n − 4]}
y(4) = 12
2y + 10 = 12
8. [Ans. D]
2y = 2
y(t) ⇒ x(t) ∗ h(t)
y=1
y(t) ⇒ e−at u(t) ∗ e−bt u(t)
y(3) = 2x + 2y + 15
If a = 5 and b = 2

info@thegateacademy.com ©Copyright reserved. Web:www.thegateacademy.com 34


Linear Time Invariant (LTI) Systems

y(3) = 19 s(−t)

Flip 1
11. [Ans. *]Range: 0 to 0
Response of overall system t
h[n] = h1 [n] + {−h2 [n] ∗ h1 [n]}
= h1 [n] − {h2 [n] ∗ h1 [n]} −1
= h1 [n] − {δ[n − 1] ∗ h1 [n]}
h(t)
= h1 [n] − h1 [n − 1]
1
= 0.5δ[n − 1] + 0.7δ[n] − 0.5δ[n − 2]
2
− 0.7δ[n − 1] t
0 1 3 4
= 0.7δ[n] − 0.2δ[n − 1] − 0.5δ[n − 2] ⋯ ①
From the given problem, we have −1
h[n] = Aδ[n] + Bδ[n − 1] − Cδ[n − 2] ⋯ ② 0−1
Slope between 3 to 4 = = −1
So, by comparing equations ① and ②, we get 1

A = 0.7, B = −0.2, C = −0.5
y(t) = ∫ s(τ)h(t − τ)dτ
Thus, we obtain A + B + C = 0 −∞

y(1) = ∫ s(τ)h(1 − τ)dτ
12. [Ans. *]Range: 8.50 to 8.80 −∞
x1 (t) s(τ)
1 1

t 2
1 0 1 τ
− + 1 3 4
2 2
x2 (t)
1 −1

s(τ)h(1 − τ)
t
9 0 9 1
− +
2 2
As convolution of two unequal length
τ
rectangles is a trapezoidal signal 0 1
z(t) 1
1 y(1) = × 1 × 1 = 0.5
2

t 14. [Ans. *]Range: 1.2 to 1.2


−5 −4 0 4 5
2 y[n] = x[n] ∗ h[n]
E = ∫ |x(t)|2 dt ∞
−2 = ∑ x[k] h[n − k]
4 5
k=−∞
E = 2 [∫ (1)2 dt + ∫ (−t + 5) ∙ dt] 2

0 4
1 = ∑ (0.8)k u[k](0.4)n−k u[n − k]
= 2 [4 + ] k=−∞
3
26 u[k] = 1, for k > 0
= = 8.67 u[n − k] = 1, for n − k > 0 or k < n
3 n

so, y[n] = ∑ (0.8)k (0.4)n−k


13. [Ans. C]
k=0
h(t) = s(Tb − t) n
flip and then shift by Tb towards right = (0.4)n ∑ 2k
k=0
1 − 2n+1
= (0.4)n [ ]
1−2
= (0.4)n [2n+1 − 1]u[n]
For n = 1, y[1] = (0.4)1 [22 − 1]

info@thegateacademy.com ©Copyright reserved. Web:www.thegateacademy.com 35


Linear Time Invariant (LTI) Systems

= (0.4) × 3 = 1.2 h(t) 42


30
28
15. [Ans. *]Range: 5.1 to 5.5
f(t) z(t)

1 1

4

−1 0 +1 t 1 3 t −5 −3 0 2 4 t
y(t) y(t) = x(t) ∗ h(t)
y(t) = h(t) ∗ δ(t) = h(t)
2 y(t)|t=4.5 = 42

18. [Ans. *]Range: 1.9 to 2.1


0 2 4 t h(t) = r(t) − r(t − 2) + 2 u(t − 3)
2
2
4
2
−6 u(t − 4) + 2r(t − 4) − 2r(t − 5)
E = ∫ (x(t)) ∙ dt + ∫ (x(t)) dt h(τ)
0 2
4 16
= ×4=
3 3 4
= 5.33
2
16. [Ans. *]Range: 49.9 to 50.1
x(t) = 5 sin 2t + 8 cos 4t + 14 cos(6t + 30°)
0 2 3 4 5 τ
H(ω)
2 −2
x(τ)

ω 1
−3 0 +3
⇒ H(ω) working as a Low Pass Filter (LPF)
⇒ So it blocks frequency ω = 4 rad/ sec and 0 4 τ
ω = 6 rad/sec x(2 − τ)
∴ y(t) ⇒ 10 sin(2t)
1
(10)2
Power P ⇒
2
100
⇒ ⇒ 50 units
2 −2 0 +2 τ

17. [Ans. *]Range: 41.9 to 42.1 y(t) = ∫ h(τ) ∙ x(t − τ) ∙ dτ


−∞
h(t) = [h1 (t) + h2 (t)] ∗ [u(t) + h3 (t) ∗ h4 (t)] ∞
y(t)
= [h1 (t) + h2 (t)] ∗ [u(t) + 3 u(t − 3) ∗ 2δ(t + 1)] | = ∫ h(τ) ∙ x(2 − τ) ∙ dτ
t=2 −∞
= [h1 (t) + h2 (t)] ∗ [u(t) + 6 u(t − 2)] ⇓
[4δ(t + 5) + 2δ(t − 2)] ∗ [u(t) + 6 u(t − 2)] h(τ) ∙ x(2 − τ)

Let z(t) h(τ) ∗ (2 − τ)


= z(t) ∗ u(t) + z(t) ∗ 6 u(t + 2)
= [4δ(t + 5) + 2δ(t − 2)] ∗ u(t) 2
+[4δ(t + 5) + 2δ(t − 2)] ∗ 6 u(t − 2)
= 4 u(t + 5) + 2 u(t − 2) + 24 u(t + 3)
0 2 τ
+ 12 u(t − 4)
1
h(t) = 4 u(t + 5) + 24 u(t + 3) + 2 u (t − 2) y(t) = × 2 × 2 = 2
2
+ 12 u(t − 4)

info@thegateacademy.com ©Copyright reserved. Web:www.thegateacademy.com 36


Linear Time Invariant (LTI) Systems

19. [Ans. C]
y(t) = x(t) ∗ n(t)
y(t) = x(t) ∗ u(t − 2)
dy(t)
= x(t) ∗ δ(t − 2)
dt
dy(t)
= x(t − 2)
dt
t
y(t) = ∫ x(t − 2) ∙ dt
−∞
y(t)
11 15
10
8 6
3

−3 −1 1 3 5 6 8 t
Area of y(t) = 16 + 20 + 22 + 30 + 3 + 12
⇒ A = 103
Energy of y(t)
⇒ 128 + 200 + 242 + 450 + 9 + 72
⇒ E = 1101

20. [Ans. B]
h(t) = (h1 (t) + h2 (t)) ∗ h3 (t)
= [2δ(t + 3) + 3δ(t − 1) + δ(t + 2)] ∗ 4u(t − 3)
= [8u(t − 3) ∗ δ(t + 3) + 12u(t − 3)
∗ δ(t − 1) + 4u(t − 3) ∗ δ(t + 2)
⇒ h(t) = [8u(t) + 12u(t − 4) + 4u(t − 1)]
h(t)

24
12
8
t
0 1 4
h(t)|t=5 = 24
T
1 +2
Power = P = lim ∫ |x(t)|2 dt
T→∞ T −T
2
T
1 1 4
2
⇒ lim [∫ (8)2 dt + ∫ (12)2 dt + ∫ (24)2 dt]
T→∞ T 0
1 4
T
1
⇒ lim [64 + 144(4 − 1) + (24)2 [t]42 ]
T→∞ T
1 T
⇒ lim (0 + 0 + (24)2 [ − 4])
T→∞ T 2
(24)2
⇒ ⇒ 288
2

info@thegateacademy.com ©Copyright reserved. Web:www.thegateacademy.com 37

You might also like